subject
Mathematics, 18.09.2019 08:30 zarakanchi

Abox plot was made to represent the number of matches won by 14 participants in a tennis tournament. the box plot had the box shifted to the right so that the left tail was much longer than the right tail. based on the box plot, which conclusion is correct?
(a)the mean and median of matches won are equal.
(b)the mean of matches won is more than the median of matches won.
(c)most of the participants won many matches, but there were some participants who won very few matches compared to the others.
(d)most of the participants won very few matches, but there were some participants who won many matches compared to the others.

ansver
Answers: 1

Another question on Mathematics

question
Mathematics, 20.06.2019 18:04
Find the third,fifth and seventh term sequence described by each rule a(n)=n+1
Answers: 2
question
Mathematics, 21.06.2019 17:00
Joseph haydn pet store offers wide variety of leashes for dogs. the store purchases one popular type of leash from its manufacturer for $4.38 each. joseph haydn pets marks up leashes at a rate of 238% of cost. what is the retail price of the popular leash?
Answers: 1
question
Mathematics, 21.06.2019 17:00
Scarlet bought three pairs of sunglasses and two shirts for $81 and paula bought one pair of sunglasses and five shirts for $105 what is the cost of one pair of sunglasses and one shirt?
Answers: 2
question
Mathematics, 21.06.2019 22:10
Which of the following circles have their centers in the third quadrant?
Answers: 2
You know the right answer?
Abox plot was made to represent the number of matches won by 14 participants in a tennis tournament....
Questions
question
Mathematics, 09.04.2020 22:01
question
Mathematics, 09.04.2020 22:01
question
Mathematics, 09.04.2020 22:02
question
Mathematics, 09.04.2020 22:02